Calculation of Large Exponents Modulo a Prime

Click For Summary
The discussion centers on calculating large exponents modulo a prime using Euler's Theorem, specifically addressing the case when the exponent is smaller than the totient function (p-1). The example provided, 95^{65} mod 131, illustrates the challenge of reducing the exponent effectively. A suggested approach involves recognizing that 95^{130} is congruent to 1 mod 131, allowing for the use of properties of exponents to simplify calculations. The conversation also hints at factoring the base for easier computations and emphasizes the iterative nature of the method. Overall, the key takeaway is the importance of leveraging modular arithmetic properties to handle smaller exponents.
Vespero
Messages
26
Reaction score
0

Homework Statement



In my Number Theory class, we learned how to calculate the value of large exponents modulo primes using Euler's Theorem. I understand how to do this with exponents larger than the value of the totient function of the prime, which is p-1, but what about when the exponent is actually smaller than that value? For example, if I have 95^{65}\ mod\ 131, I don't see how I can reduce the exponent the way I would for something like 95^{261} \equiv 95^{2(130)+1} \equiv 95\ (mod\ 131).

Homework Equations



x^{p-1} \equiv 1\ (mod\ p)

and the more general form for a composite number. Also, any basic properties of exponents and moduli that I'm missing.

The Attempt at a Solution



I'm just not sure where to begin. Obviously, 65 is half of 130, and we know that 95^{130} \equiv 1\ (mod\ 131), but how can I use that?
 
Physics news on Phys.org
Vespero said:

Homework Statement



In my Number Theory class, we learned how to calculate the value of large exponents modulo primes using Euler's Theorem. I understand how to do this with exponents larger than the value of the totient function of the prime, which is p-1, but what about when the exponent is actually smaller than that value? For example, if I have 95^{65}\ mod\ 131, I don't see how I can reduce the exponent the way I would for something like 95^{261} \equiv 95^{2(130)+1} \equiv 95\ (mod\ 131).



Homework Equations



x^{p-1} \equiv 1\ (mod\ p)

and the more general form for a composite number. Also, any basic properties of exponents and moduli that I'm missing.



The Attempt at a Solution



I'm just not sure where to begin. Obviously, 65 is half of 130, and we know that 95^{130} \equiv 1\ (mod\ 131), but how can I use that?
You have pretty nearly answered your own question.

You have that 95^{65\cdot2}\equiv1 (\text{mod} 131)\,. Then 95^{65\cdot2} - 1\equiv0 (\text{mod} 131)\,.

Use the fact that 95^{65\cdot2}=(95^ {65})^2\,.

Factor the difference of squares and see where that leads you.
 
Hi Vespero! :smile:

The generic method is to do this:
95^{65} \equiv 95^{2\cdot 32+1} \equiv (95^2)^{32} \cdot 95 \equiv (95^2 \textrm{ mod } 131)^{32} \cdot 95 (\textrm{mod } 131)
Repeat, until you are done.

If you want, you can factor 95 first into 5 x 19 to get easier calculations.

This is quite a bit of work, although you can recycle some of your work.

@SammyS: I believe your method leads to 2 solutions...
 
Question: A clock's minute hand has length 4 and its hour hand has length 3. What is the distance between the tips at the moment when it is increasing most rapidly?(Putnam Exam Question) Answer: Making assumption that both the hands moves at constant angular velocities, the answer is ## \sqrt{7} .## But don't you think this assumption is somewhat doubtful and wrong?

Similar threads

Replies
4
Views
1K
  • · Replies 5 ·
Replies
5
Views
1K
Replies
4
Views
2K
  • · Replies 6 ·
Replies
6
Views
2K
  • · Replies 16 ·
Replies
16
Views
3K
  • · Replies 16 ·
Replies
16
Views
2K
  • · Replies 6 ·
Replies
6
Views
2K
  • · Replies 11 ·
Replies
11
Views
2K
  • · Replies 9 ·
Replies
9
Views
3K
  • · Replies 2 ·
Replies
2
Views
2K